Howdy, Stranger!

It looks like you're new here. If you want to get involved, click one of these buttons!

PT 7.4.11 Scientific research at a certain university was supported in part by an annual grant..

gerstle19gerstle19 Core Member

Can anyone explain why C would be incorrect, and why A would instead be the correct AC?

Comments

  • Chris NguyenChris Nguyen Alum Member Administrator Sage 7Sage Tutor
    4593 karma

    Hey there!

    So essentially, the stimulus says we have a university that's participating in weapons research. Because of this, the grant org is threatening to take away their funding. The university responds by promising the grant money will not be spent on weapons research. From this, the grant org concludes that the grant funding will not benefit the university's weapons research.

    This argument is flawed because there is a possibility the grant money can still indirectly benefit weapons research. The addition of grant money has the possibility of freeing up extra money that can be be allocated towards weapons research, therefore aiding weapons research. If the grant money is withdrawn, the university may have to withdraw funds from weapons research and put that money where the grant money was. Say you have $5 in weapons funding and $5 in humanitarian funding. Then this grant comes along with $5 in humanitarian funding. Then you're like "oh awesome, now I can just put that grant money in humanitarian funding, and move the original humanitarian funds to weapons funding. Now you have $10 in weapons funding instead of $5. Your grant funds are aiding weapons research!

    A) States exactly this. The argument doesn't take into account the possibility the grant money can allow the university to add more funds to weapons research, therefore aiding weapons research.

    C) Is true, but is not a flaw in reasoning. Sure, the argument overlooks the reason why the university made its promise, but that point is irrelevant in determining what the flaw is in the argument's reasoning. Why does it matter why the university made its promise? We are looking at the flaw in the premise/conclusion relationship in the stimulus. This doesn't give us that. The argument overlooks a lot of things. It also overlooks the reason the grant org is against weapon's research, but just like (C), it's not a flaw in the argument's reasoning, and is incorrect because of that.

  • gerstle19gerstle19 Core Member
    100 karma

    @Christopherr Thanks so much for the thorough response, it's a tremendous help!

Sign In or Register to comment.